ความสามารถในการละลายของการเติมเมทริกซ์


11

เมทริกซ์มีขนาดn × n ( n - 1 ) เราต้องการเติมAโดยใช้จำนวนเต็มระหว่าง1ถึงnรวมAn×n(n1)A1n

ที่ต้องการ:

  1. คอลัมน์ของแต่ละคือการเปลี่ยนแปลงของ1 , ... , nA1,,n
  2. submatrix ใด ๆ ที่เกิดขึ้นจากแถวสองแถวของไม่สามารถมีคอลัมน์ที่เหมือนกันได้A

คำถาม:

เป็นไปได้หรือไม่ที่จะเติมเมทริกซ์ให้เป็นไปตามข้อกำหนด?

ความสัมพันธ์กับการเข้ารหัส:

หมายเลขแถวแต่ละหมายเลขสอดคล้องกับข้อความธรรมดา แต่ละคอลัมน์สอดคล้องกับคีย์ เนื่องจากคีย์กำหนดการฉีดแต่ละคอลัมน์จะต้องมีการเปลี่ยนแปลง ข้อกำหนดที่สองมีไว้เพื่อความลับที่สมบูรณ์แบบสำหรับสองข้อความ


1
เนื่องจากคุณติดแท็กสิ่งนี้ด้วยความปลอดภัย cr.crypto มันจะปรับปรุงคำถามหากคุณสามารถระบุว่าเกี่ยวข้องกับ crypto / ความปลอดภัยหรือไม่
Dave Clarke

1
การสังเกตอย่างง่าย: เมทริกซ์นั้นมีอยู่สำหรับn≤4 สำหรับn≤3ใช้การเปลี่ยนลำดับทั้งหมด สำหรับ n = 4 ทางแก้ปัญหาเดียวคือการเปลี่ยนลำดับทั้งหมดหรือแม้แต่การเปลี่ยนลำดับคี่ทั้งหมด
Tsuyoshi Ito

ขอบคุณ Ito ที่จริงฉันคิดคำตอบเมื่อด้วยมือ แต่กลายเป็นสิ่งที่ยากมากขึ้นเมื่อn 5 การระเบิดแบบเลขชี้กำลังเกิดขึ้น n4n5
Cyker

3
(1) ฉันคิดว่าปัญหาเกี่ยวข้องกับทฤษฎีการเข้ารหัสและเพิ่มเป็นแท็ก (2) การสังเกตอื่น: ปัญหาสามารถระบุได้ดังนี้ ค้นหาเมทริกซ์ B ที่มีขนาด n × (n ^ 2) เพื่อให้คอลัมน์ n แรกของ B แต่ละรายการซ้ำซ้อนของจำนวนเดียวกันและ B นั้นเป็นไปตามเงื่อนไข 2 ในคำถาม หาก B นั้นมีอยู่แล้วคอลัมน์ n (n − 1) สุดท้ายแต่ละอันของ B จะต้องเป็นแบบเรียงสับเปลี่ยน ในทางกลับกันเมทริกซ์ใด ๆ A ที่เป็นไปตามเงื่อนไขที่ 1 และ 2 สามารถแปลงเป็นเมทริกซ์ B โดยการแนบคอลัมน์ที่ระบุ n ไปทางซ้ายของ A.
Tsuyoshi Ito

คำตอบ:


11

Tsuyoshi การสังเกตที่ยอดเยี่ยมในความคิดเห็นของคุณ! ฉันคิดว่านี่เกือบจะแก้ปัญหาได้

พิจารณาคำถามสองข้อต่อไปนี้

  1. มีแถวที่มีความยาวแถว n ( n - 1 ) หรือไม่ดังนั้นจึงไม่มีตัวเลขปรากฏในคอลัมน์ใด ๆ สองครั้งและสำหรับแต่ละแถวของคู่ที่เรียงลำดับทั้งหมดที่กำหนดโดยคอลัมน์นั้นแตกต่างกันหรือไม่?kn(n-1)
  2. มีแถวที่มีความยาว n n 2เพื่อให้สำหรับแต่ละแถวแต่ละคู่คู่ที่สั่งซื้อทั้งหมดที่กำหนดโดยคอลัมน์นั้นแตกต่างกันหรือไม่?kn2

การสังเกตของ Tsuyoshi ในความคิดเห็นของเขาแสดงให้เห็นว่าหากคุณสามารถบรรลุค่าสำหรับคำถาม (1) คุณสามารถบรรลุค่าkสำหรับคำถาม (2) ตอนนี้เราแสดงให้เห็นว่าหากเราสามารถบรรลุคุณค่าของkสำหรับคำถาม (2) เราสามารถทำให้ได้ค่าk - 1สำหรับคำถาม (1) ดังนั้นคำตอบของคำถามทั้งสองนี้ก็เกือบจะเหมือนกันkkkk-1

การก่อสร้างไปดังต่อไปนี้: ละเว้นแถวแรกยกเว้นใส่ทั้งหมดเป็นครั้งแรกในnตำแหน่ง ตอนนี้คุณสามารถใช้การเปลี่ยนแปลงค่า{ 1 , 2 , , n }กับแต่ละแถวที่เหลือk - 1ดังนั้นยกเว้นรายการแรกคอลัมน์nคอลัมน์แรกแต่ละคอลัมน์มีค่าเหมือนกันและโดยการสังเกตของ Tsuyoshi ในความคิดเห็นสิ่งนี้จะช่วยให้คุณมีชุดของk - 1แถวที่ตรงกับเงื่อนไขของคุณ1n{1,2,...,n}k-1nk-1

ตอนนี้ถ้าคุณมีชุดของแถวยาวn 2กับคู่ของแถวที่มีคู่สั่งซื้อทั้งหมดในแต่ละทุกคอลัมน์แล้วนี้จะเทียบเท่ากับชุดของk - 2สี่เหลี่ยมมุมฉากละติน แต่ละแถว3 , 4 , , kให้ตารางละติน หากต้องการให้ละตินสแควร์เชื่อมโยงกับแถวjให้ใส่ค่าในคอลัมน์iของแถวjในเซลล์ที่มีพิกัดที่กำหนดโดยคู่ที่สั่งในคอลัมน์iในแถวสองแถวแรกkn2k-2 34...kJผมJผม

ถ้าไม่ได้เป็นอำนาจนายกกี่ฉากร่วมกันสี่เหลี่ยมละตินของการสั่งซื้อnมีอยู่เป็นปัญหาเปิดที่มีชื่อเสียงและผมไม่เชื่อว่าชุดใด ๆ ของn - 2สี่เหลี่ยมมุมฉากละตินเป็นที่รู้จักกันอยู่สำหรับnไม่ได้เป็นอำนาจนายก; ฉันทามติทั่วไปคือชุดดังกล่าวไม่อยู่ เพียงผลการพิสูจน์เพื่อให้ห่างไกลคือว่าชุดดังกล่าวไม่ได้มีอยู่สำหรับn = 6 สิ่งที่เป็นที่รู้จักคือจำนวนkของแถวที่เป็นไปได้จะเพิ่มขึ้นอย่างน้อยเท่ากับk = Ω ( n c )สำหรับบางnnn-2nn=6kk=Ω(n). ฉันเชื่อว่ามีออร์เดอร์สี่เหลี่ยมจตุรัสละติน 8 มุมฉากของ 10 คำสั่งที่ยังเปิดอยู่ (เป็นที่รู้กันว่าไม่มี 9 แต่เนื่องจากความแตกต่างที่เป็นไปได้ของในคำตอบของคำถามสองข้อนี้ไม่ได้บอกอะไรเราเกี่ยวกับปัญหาดั้งเดิม)1

สำหรับ , kสูงสุดที่คุณจะได้คือ 3, และปรากฎว่าคุณสามารถได้รับสามแถวสำหรับปัญหา (1) โดยดูตารางละติน6 × 6ใด ๆ ที่มีการตัดขวางซึ่งมีตัวอย่างที่ไม่เทียบเท่าจำนวนมาก . สำหรับn = 10มีสิ่งปลูกสร้างที่รู้จักกันให้สองช่องสี่เหลี่ยมละตินมุมฉาก หากสี่เหลี่ยมเหล่านี้มีการตัดขวางทั่วไปคุณจะได้รับk = 4สำหรับปัญหา (1)n=6k6×6n=10k=4


ขอบคุณสำหรับการวิเคราะห์โดยละเอียดศ. ดังนั้นจากเหตุผลนี้ 1) ถ้ามีไม่ได้อยู่ชุดของ Mols เราสามารถยืนยันปัญหาเดิมไม่เป็นไปได้สำหรับn 2) ถ้ามีชุดของn - 1 Mols เราสามารถยืนยันปัญหาเดิมเป็นไปได้สำหรับn เนื่องจากเมื่อnเป็นมหาอำนาจจึงมีชุดของn - 1 MOLS ซึ่งจะให้มุมมองทางเลือกของโซลูชันบางส่วนของ Ito และเราได้พบปัญหาเดิมไม่เป็นไปได้เมื่อn = 6 น่าชื่นชมจริงๆ! n-2nn-1nnn-1n=6
Cyker

นี่คือการเชื่อมต่อที่ดีมาก ขอบคุณสำหรับคำตอบ! จุดเล็ก ๆ น้อย ๆ : ตามวิกิพีเดียเป็นที่รู้กันว่า n − 1 สี่เหลี่ยมจตุรัสลาติน orthogonal นั้นมีอยู่สำหรับ n ไพรเวทพาวเวอร์ไม่ใช่เฉพาะสำหรับ n ไพรม์
Tsuyoshi Ito

@Tsuyoshi - โอ๊ะโอ ฉันรู้ว่า; ฉันแค่บอกว่ามันผิด การก่อสร้างมาจากทุ่ง จำกัด ขอบคุณสำหรับการแก้ไข แก้ไขได้ทันที
Peter Shor

ฉันเดาอย่างนั้น :)
Tsuyoshi Ito

11

นี่เป็นวิธีแก้ปัญหาบางส่วน เมทริกซ์นั้นมีอยู่ถ้าnเป็นพลังพิเศษ

ให้Fเป็นฟิลด์ จำกัด ของการสั่งซื้อn เราสร้างเมทริกซ์n × n ( n −1) ซึ่งมีแถวที่มีป้ายกำกับโดยFซึ่งคอลัมน์จะมีป้ายกำกับโดย ( F ∖ {0}) × Fและรายการที่อยู่ในFดังนี้: i -th แถวของ คอลัมน์ ( , ) จะได้รับจากไอ + ข ในคำแต่ละสอดคล้องคอลัมน์ในระดับหนึ่งพหุนามในF จากนั้นแต่ละคอลัมน์จะมีองค์ประกอบของFแต่ละองค์ประกอบ แน่นอนหนึ่งครั้งและไม่มีสองคอลัมน์ที่มีรายการเท่ากันในมากกว่าหนึ่งแถวเนื่องจากค่าของชื่อพหุนามดีกรีหนึ่งที่แตกต่างกันสองอันสามารถตรงกันได้มากที่สุดในจุดเดียว

(หากคุณต้องการเมทริกซ์ที่มีรายการอยู่ใน {1, …, n } แทนที่จะเป็นในFให้แทนที่องค์ประกอบของFด้วย {1, …, n } โดยพลการ)


n+1

@ บทความ: อาจมีโดยเฉพาะคำตอบของปีเตอร์เชื่อมต่อคำถามนี้กับสี่เหลี่ยมละตินมุมฉาก (ข้อจำกัดความรับผิดชอบ: ในมุมมองที่ไม่ใช่ผู้เชี่ยวชาญของฉันสี่เหลี่ยมละตินมุมฉาก, MUBs, การออกแบบ combinatorial, การออกแบบที่รวมกันและ SIC-POVMs แทบจะแยกไม่ออก)
Tsuyoshi Ito

ขอบคุณมากอิโตะ! การออกแบบนี้ดูสวยงามจริงๆ!
Cyker
โดยการใช้ไซต์ของเรา หมายความว่าคุณได้อ่านและทำความเข้าใจนโยบายคุกกี้และนโยบายความเป็นส่วนตัวของเราแล้ว
Licensed under cc by-sa 3.0 with attribution required.